You are on page 1of 5

Homework

2013-7-16
Problem 1. Find all the invertible elements of

Solution: We want to nd elements a Z24 with the property that for an element b Z24 , a b = 1.

Z24 .

Z24 = 0, 1, 2, 3, ..., 23
. We showed in class that if for a Zn , b Zn : a b = 0, a cant have an inverse. Any of the numbers in this set with two as a factor cant have inverses because (2n)(12) = 0. The numbers with three as a factor cant have inverses because (3n)(8) = 0. The elements in Z24 that dont have neither two nor three as a factor are: 1, 4, 7, 11, 13, 17 and 19. Now we will try to nd inverses for these elements. 11=11 5 5 = 25 1 7 7 = 49 1 11 11 = 121 1 13 13 = 169 1 17 17 = 289 1 1

19 19 = 361 1 23 23 = 529 1 the set of invertible elements of

Z24 is U24 = {1, 5, 7, 11, 13, 17, 19, 23}


in the ring (Zn , , , [0],

Problem 2. Prove that the operations and [1]) are well dened.

Solution: In this ring, [a]+[b] = [a + b] and [a][b] = [ab]. To show that these operations are well dened we need to show that if [a] = [a ] and [b] = [b ], then [a + b] = [a + b ] and [ab] = [a b ]. Suppose that [a] = [a ] and [b] = [b ]. Then n|a a and n|b b . From this we get that n|(a a ) + (b b ) = (a + b) (a + b ) (a + b) (a + b ) (mod n) so [a + b] = [a + b ]. This shows that addition is well dened. Now, from n|a a and n|b b , we get n|(a a )(b b ) = ab + a b a b ab = (ab + a b ) (a b + ab ) (ab + a b ) (a b + ab ) (mod n). ab + a b (a b + ab ) ab + a b 2a b a b + ab 2a b ab a b a b a b + ab a b a (b b ) + b (a a ) Since n|(b b ) and n|(a a ), we get ab a b 0(mod n) ab a b (modn) [ab] = [a b ] multiplication is well dened.

For problems 3-4, let a = 1211 and b = 6203.


Problem 3. Find gcd(a, b). Solution: gcd(1211, 6203) 6203 = 5(1211) + 148

1211 = 8(148) + 27 148 = 5(27) + 13 27 = 2(13) + 1 13 = 13(1) + 0

gcd(1211, 6203) = 1 Problem 4. Find s and t such that s(1211) + t(6203) = gcd(1211, 6203). Solution: 1 = 27 2(13)) = 27 2(148 5(27)) = 27 2(148) + 10(27) = 11(27) 2(148) = 11(1211 8(148)) 2(148) = 11(1211) 88(148) 2(148) = 11(1211) 90(148) = 11(1211) 90(6203 5(1211)) = 11(1211) 90(6203) + 450(1211) = 461(1211) 90(6203) s = 461 and t = 90 Problem 5. Show that if (F, +, , 0, 1) is a eld with the property that a2 + b2 = 0, for a, b F, then R={matrices of the form {{a,b},{-b,a}}: a, b F} is a eld.

Solution: We want to show that if (F, +, , 0, 1) is a eld with the property a b that a2 + b2 = 0 for a, b F, then R = : a, b F is a eld. b a We know that the matrices form a ring R will be associative and commutative under +. It will also be associative under and the multiplication 3

will distribute under the sum. 0 0 it is in R. The 1 matrix can be 0 0 1 0 a b written as it is in R. The additive inverse of an element 0 1 b a a b is which is in R; c R, c R. b a The 0 matrix can be written as Now, to show that R is a eld, we only need to show that it is closed under both operations and that it is commutative under . a b c d Let x := and y := . Now, b a d c x+y = a+c b+d (a + c) (b + d) = x+y R b d a + c (b + d) (a + c)

R is closed under +. Now we will show that x y R (ac bd) (ad + bc) a b xy = . Since x y is of the form , (ad + bc) (ac bd) b a x y R R is closed under . To conclude with this problem, we will show that x y = y x. We have xy = and (ca db) (cb + da) (ac bd) (ad + bc) = =xy (da cb) (db + ca) (ad + bc) (ac bd) (ac bd) (ad + bc) (ad + bc) (ac bd)

yx=

if (F, +, , 0, 1) is a eld with the property that a2 + b2 = 0 for a, b F, a b then R = : a, b F is a eld. b a

Problem 6. Construct a eld of 9 elements. Write the elemnts of the eld explicitely. Solution: Using the fact that if (F, +, , 0, 1) is a eld with the property a b that a2 + b2 = 0 for a, b F, then R = : a, b F is a eld, we b a can construct a eld of 9 elements. Since 3 is prime, Z3 is a eld. To construct a eld with Z3 in the way explained above, we need to show that in Z3 , a2 + b2 = 0. To do this, well show that for every prime p of the form 4k + 3, the eld Zp has no elememts with the proberty that x2 + y 2 0 (mod p). Suppose there exists x, y Zp such that x2 + y 2 0 (mod p). This is equivalent to x2 (y 2 ) (mod p). Raising this equivalence to the power (p1)/2, we get (x2 )(p1)/2 ((y 2 )(p1)/2 ) (mod p). Since p is of the form 4k + 3, (p 1)/2 is of the form 2k + 1. Therefore, we get xp1 (y p1 ). By Fermats Little Theorem, xp1 1 (mod p). Then we have that (y p1 ) 1 (mod p) which implies that y p1 1 (mod p) but by Fermats Little Theorem y p1 (mod p) 1, a contradiction if p is of the form 4k + 3, for a, b Zp , a2 + b2 = 0. we have that

F9 =

1 1 1 2 2 0 2 1 2 2 0 0 0 1 0 2 1 0 , , , , , , , , 0 0 2 0 1 0 0 1 2 1 1 1 0 2 2 2 1 2

Problem 7. Let n > 0, a > 0, a Zn : gcd(a, n) > 1. Show that a1 does not exist in Zn . Solution: If there exists x Zn such that ax 0 then a1 Zn . Let gcd(a, n) = d > 1. Then a = dk and n = dl with k, l N and k < l < n. Since l < n, l Zn . Then we take the product of a and l and we get that al = dkl = k (dl) = k (n) 0 a1 does not exist in

Zn

You might also like